From 46562ba25383fc65407a383f1d05b260d1297162 Mon Sep 17 00:00:00 2001 From: "W. Trevor King" Date: Thu, 31 May 2012 08:44:20 -0400 Subject: [PATCH] Add missing figures and solutions to Serway & Jewett v8's chapter 29. --- .../Serway_and_Jewett_8/problem29.02.tex | 28 ++++-- .../Serway_and_Jewett_8/problem29.03.tex | 1 + .../Serway_and_Jewett_8/problem29.22.tex | 29 +++---- .../Serway_and_Jewett_8/problem29.31.tex | 18 ++++ .../Serway_and_Jewett_8/problem29.37.tex | 54 ++++++++++++ .../Serway_and_Jewett_8/problem29.40.tex | 77 +++++++++++++++- .../Serway_and_Jewett_8/problem29.57.tex | 87 ++++++++++++++++++- .../Serway_and_Jewett_8/problem29.61.tex | 37 ++++++++ 8 files changed, 305 insertions(+), 26 deletions(-) diff --git a/latex/problems/Serway_and_Jewett_8/problem29.02.tex b/latex/problems/Serway_and_Jewett_8/problem29.02.tex index 6c133b6..c54394d 100644 --- a/latex/problems/Serway_and_Jewett_8/problem29.02.tex +++ b/latex/problems/Serway_and_Jewett_8/problem29.02.tex @@ -1,4 +1,4 @@ -\begin{problem*}{29.3} +\begin{problem*}{29.2} Determine the initial direction of the deflection of charged particles as they enter the magnetic fields shown in Figure~P29.2. \begin{center} @@ -32,7 +32,7 @@ Vector v = Velocity(a.center(), dir=0); v.draw(); a.draw(); -label("\Part{a}", (0,0.5*height), N); +label("\Part{a}", (0,-0.5*height), S); \end{asy} \hspace{\stretch{1}} \begin{asy} @@ -60,7 +60,7 @@ Vector v = Velocity(a.center(), dir=180); v.draw(); a.draw(); -label("\Part{b}", (0,0.5*height), N); +label("\Part{b}", (0,-0.5*height), S); \end{asy} \hspace{\stretch{1}} \begin{asy} @@ -89,7 +89,7 @@ Vector v = Velocity(a.center(), dir=180); v.draw(); a.draw(); -label("\Part{c}", (0,0.5*height), N); +label("\Part{c}", (0,-0.5*height), S); \end{asy} \hspace{\stretch{1}} \begin{asy} @@ -126,11 +126,29 @@ Vector v = Velocity(a.center(), dir=90); v.draw(); a.draw(); -label("\Part{d}", (0,0.5*height), N); +label("\Part{d}", a.lc.center, S); \end{asy} \hspace{\stretch{1}} +\rule{0pt}{0pt} \end{center} \end{problem*} \begin{solution} +The force on a charged particle moving through a magnetic field is +$\vect{F}=q\vect{v}\times\vect{B}$. From Newton second law, +$\vect{a}=\vect{F}/m=q/m\cdot\vect{v}\times\vect{B}$. We can find the +direction of deflection comes using this formula and the right hand +rule. + +\Part{a} +\ans{Up}. + +\Part{b} +\ans{Out of the page}. + +\Part{c} +\ans{No deflection}. + +\Part{d} +\ans{Into the page}. \end{solution} diff --git a/latex/problems/Serway_and_Jewett_8/problem29.03.tex b/latex/problems/Serway_and_Jewett_8/problem29.03.tex index b43eab8..fd89e83 100644 --- a/latex/problems/Serway_and_Jewett_8/problem29.03.tex +++ b/latex/problems/Serway_and_Jewett_8/problem29.03.tex @@ -41,6 +41,7 @@ v.draw(); label("\Part{c}", (0,0), S); \end{asy} \hspace{\stretch{1}} +\rule{0pt}{0pt} \end{center} \end{problem*} diff --git a/latex/problems/Serway_and_Jewett_8/problem29.22.tex b/latex/problems/Serway_and_Jewett_8/problem29.22.tex index 1831a54..bc2162c 100644 --- a/latex/problems/Serway_and_Jewett_8/problem29.22.tex +++ b/latex/problems/Serway_and_Jewett_8/problem29.22.tex @@ -1,7 +1,7 @@ \begin{problem*}{29.22} Assume the region to the right of a certain plain contains a uniform magnetic field of magnitude $1.00\U{mT}$ and the field is zero in the -fregion to the left of the plane as shown in Figure~P29.22. An +region to the left of the plane as shown in Figure~P29.22. An electron, originally traveling perpendicular to the boundary plane, passes into the region of the field. \Part{a} Determine the time interval required for the electron to leave the ``field-filled'' @@ -19,21 +19,10 @@ real width = 2u; real dx = 0.49u; real dy = dx; -Vector Bs[]; -int n = (int)(width / dx); -int m = (int)(height / dy); -real xstart = -width/2 + (dx+fmod(width,dx))/2.0; -real ystart = -height/2 + (dy+fmod(height,dy))/2.0; -for (int i=0; i| +% +-------------+ +% Bout +% +\begin{asy} +import Mechanics; +import ElectroMag; +import Circ; + +real u = 1cm; + +MultiTerminal Bat = battery(dir=180, label=Label("$24.0\U{V}$", align=N)); +MultiTerminal R = resistor(Bat.terminal[1], dir=180); +Spring Sl = Spring(R.terminal[1] - (0,u), R.terminal[1] - (0,2u), + deadLength=1mm, unstretchedLength=u, L=Label("$k$", embed=Shift)); +Sl.draw(); +Spring Sr = Spring(Bat.terminal[0] - (0,u), Bat.terminal[0] - (0,2u), + deadLength=1mm, unstretchedLength=u, L=Label("$k$", embed=Shift)); +Sr.draw(); + +Vector v = BField(phi=90); +real w = Sr.pTo.x - Sl.pTo.x; +vector_field((Sl.pTo + Sr.pTo)/2 + (0, -u), width=1.5*w, height=u, v=v); + +Distance DL = Distance(Sl.pTo, Sr.pTo, offset=u/3, + Label("$5.00\U{cm}$", align=N)); +DL.draw(); + +wire(R.terminal[1], Sl.pFrom); +wire(Bat.terminal[0], Sr.pFrom); +wire(Sl.pTo, Sr.pTo, udsq, dist=-u); +\end{asy} +\end{center} \end{problem*} \begin{solution} +We have two situations here, one where $B=0$ and $\Delta +x=0.500\U{cm}$ (call this $B_1$ and $\Delta x_1$), and another where +$B$ is unknown and $\Delta x=\Delta x_1 + 0.300\U{cm}=0.800\U{cm}$ +(call this $B_2$ and $\Delta x_2$). Because both are in equilibrium, +the net force on the hanging wire must be zero in each case. The only +forces acting on the wire are gravity pulling down with $F_g=mg$, two +springs pulling up with $F_s=k\Delta x$ (for each spring), and the magnetic +field pulling down with $F_B=ILB\cos(90\dg)$. Summing forces in the +vertical direction for the first situation, +\begin{align} + 0 &= \sum_i F_{i,y} = 2k\Delta x_1 - mg \\ + 2k\Delta x_1 &= mg \\ + k &= \frac{mg}{2\Delta x_1} \;. +\end{align} +We can plug this into the sum of vertical forces for the second +situation, +\begin{align} + 0 &= \sum_i F_{i,y} = 2k\Delta x_2 - mg - ILB_2 \\ + ILB_2 &= 2k\Delta x_2 - mg + = 2\frac{mg}{2\Delta x_1}\Delta x_2 - mg + = mg\p({\frac{\Delta x_2}{\Delta x_1} - 1}) \\ + B_2 &= \frac{mg}{IL}\p({\frac{\Delta x_2}{\Delta x_1} - 1}) \;. +\end{align} +This looks good, but we don't have a value for the current $I$. Using +a Kirchhoff loop starting behind the battery and going counter +clockwise, +\begin{align} + 0 &= V - IR \\ + I &= \frac{V}{R} \;. +\end{align} +Plugging in for $B_2$, +\begin{equation} + B_2 = \frac{mgR}{VL}\p({\frac{\Delta x_2}{\Delta x_1} - 1}) + = \frac{0.0100\U{kg} \cdot 9.80\U{m/s$^2$} \cdot 12.0\U{\Ohm}} + {24.0\U{V} \cdot 0.0500\U{m}} + \p({\frac{0.800\U{cm}}{0.500\U{cm}} - 1}) + = \ans{588\U{mT}} \;. +\end{equation} \end{solution} diff --git a/latex/problems/Serway_and_Jewett_8/problem29.61.tex b/latex/problems/Serway_and_Jewett_8/problem29.61.tex index 6c1cf45..97a1358 100644 --- a/latex/problems/Serway_and_Jewett_8/problem29.61.tex +++ b/latex/problems/Serway_and_Jewett_8/problem29.61.tex @@ -6,4 +6,41 @@ field is required to keep the rod moving at a constant speed? \end{problem*} \begin{solution} +This is just like Problem~29.40, except the force balancing the +magnetic force is friction instead of gravity. Drawing a free-body +diagram looking from the side, +\begin{center} +\begin{asy} +import Mechanics; + +real mag = 1cm; +real mu = 0.3; + +Vector F_B = Force(mag=mu*mag, Label("$F_B$", position=EndPoint)); +Vector F_g = Force(mag=mag, dir=-90, Label("$F_g$", position=EndPoint)); +Vector F_N = Force(mag=mag, dir=90, Label("$F_N$", position=EndPoint)); +Vector F_f = Force(mag=mu*mag, dir=180, Label("$F_f$", position=EndPoint)); +F_B.draw(); +F_g.draw(); +F_N.draw(); +F_f.draw(); +dot((0,0)); +\end{asy} +\end{center} + +Summing the forces in the vertical direction, +\begin{align} + 0 &= \sum_i F_{i,y} = F_N - F_g = F_N - mg \\ + F_N &= mg \\ +\end{align} +Now that we know the normal force, we can find the force of friction +and sum the forces in the horizontal direction. +\begin{align} + 0 &= \sum_i F_{i,x} = F_B - F_f = ILB - \mu F_N = ILB\sin(90\dg) - \mu mg \\ + ILB &= \mu mg \\ + B &= \frac{\mu mg}{IL} + = \frac{0.100 \cdot 0.200\U{kg} \cdot 9.80\U{m/s$^2$}} + {10.0\U{A} \cdot 0.500\U{m}} + = \ans{39.2\U{mT}} \;. +\end{align} \end{solution} -- 2.26.2